0
$\begingroup$

Let $S$ be any subset of $V^*$ for some finite dimensional space $V$. Define $\text{Ann}(S) = \{v \in V \mid f(v) = 0 \text{ for all } f \in S\}$. Let $W_{1}$ and $W_{2}$ be subspaces of $V^*$. Prove that $\text{Ann}(W_{1}+W_{2}) = \text{Ann}(W_{1}) \cap \text{Ann}(W_{2})$.
My attempt: ($\text{Ann}(W_{1}) \cap \text{Ann}(W_{2}) \subseteq \text{Ann}(W_{1}+W_{2})$): Let $w \in \text{Ann}(W_{1}) \cap \text{Ann}(W_{2})$. We need to show that $w \in \text{Ann}(W_{1}+W_{2})$. Since $w \in \text{Ann}(W_{1}) \cap \text{Ann}(W_{2})$, $w \in \text{Ann}(W_{1})$ and $w \in \text{Ann}(W_{2})$. By definition, $f_{1}(w) = 0$ for all $f_{1} \in W_{1}$ and $f_{2}(w) = 0$ for all $f_{2} \in W$. Every $f$ in $W_{1} +W_{2}$ can be written as $f = f_{1}+f_{2}$ for some $f_{1} \in W_{1}$ and $f_{2} \in W_{2}$. And note that $$(f_{1}+f_{2})(w) = f_{1}(w) +f_{2}(w) = 0.$$ This implies that $w \in \text{Ann}(W_{1}+W_{2})$.
($\text{Ann}(W_{1}+W_{2}) \subseteq \text{Ann}(W_{1}) \cap \text{Ann}(W_{2})$): We let $w \in \text{Ann}(W_{1}+W_{2})$. By definition, $f (w) = 0$ for all $f \in W_{1} +W_{2}$. And we write $f = f_{1}+f_{2}$ for some $f_{1} \in W_{1}$, $f_{2} \in W_{2}$. This means that $(f_{1}+f_{2})(w) = f_{1}(w) +f_{2}(w) = 0$. But I'm confused about how to go from here to show that $f_{1}(w) = f_{2}(w) = 0$. Please help! Thanks!

$\endgroup$
3
  • $\begingroup$ Simply, $W_1 \subseteq W_1 + W_2$ and $W_2 \subseteq W_1 + W_2$, since both contain the zero vector. And if $f(v)=0$ for all $f$ in some $A$, this is clearly also true for all subsets of $A$. $\endgroup$ Commented Apr 13 at 19:03
  • $\begingroup$ You probably meant $f_2(w)$, not $f_2(2)$. $\endgroup$ Commented Apr 13 at 20:13
  • $\begingroup$ @jjagmath Yes! Thanks for pointing it out! $\endgroup$ Commented Apr 13 at 20:26

1 Answer 1

2
$\begingroup$

Suppose that $f\notin\operatorname{Ann}(W_1)$. Then there is some $w\in W_1$ such that $f(w)\ne0$. But $w\in W_1+W_2$; so, this shows that $f\notin\operatorname{Ann}(W_1+W_2)$. By the same argument,$$f\notin\operatorname{Ann}(W_2)\implies f\notin\operatorname{Ann}(W_1+W_2).$$We deduce that$$f\in(\operatorname{Ann}(W_1))^\complement\cup(\operatorname{Ann}(W_2))^\complement\implies f\in(\operatorname{Ann}(W_1+W_2))^\complement.$$In other words,$$f\in(\operatorname{Ann}(W_1)\cap\operatorname{Ann}(W_2))^\complement\implies f\in(\operatorname{Ann}(W_1+W_2))^\complement,$$which is the same thing as asserting that$$\operatorname{Ann}(W_1+W_2)\subset\operatorname{Ann}(W_1)\cap\operatorname{Aut}(W_2).$$

$\endgroup$
4
  • $\begingroup$ The question is about $\text{Ann}$, not $\text{Aut}$. $\endgroup$ Commented Apr 13 at 20:11
  • $\begingroup$ @jjagmath It was a typo. I meant $\operatorname{Ann}$. $\endgroup$ Commented Apr 13 at 20:21
  • $\begingroup$ I'm curious. Did you really write it wrong more than 10 times or do you use some automated process to type your posts? $\endgroup$ Commented Apr 13 at 21:58
  • $\begingroup$ @jjagmath The first time, I wrote \operatorname{Aut} and then I did COPY + PASTE for all other instances. $\endgroup$ Commented Apr 13 at 22:02

You must log in to answer this question.

Start asking to get answers

Find the answer to your question by asking.

Ask question

Explore related questions

See similar questions with these tags.